Республиканская олимпиада по математике, 2023 год, 9 класс


Решите уравнение в простых числах $p^3 + q^3 + r^3 = p^2qr.$ ( Ануарбеков Т. )
посмотреть в олимпиаде

Комментарий/решение:

пред. Правка 4   1
2023-03-24 03:44:10.0 #

С начало допустим что хотя бы двое из тройки равны отсюда очевидно следует что они все равны 3 тогда допусти все числа попарно взаимнопростые тогда заметим что очевидно $p^2$ |($q^3+r^3$) тогда допустим p= 3 тогда Б.О.О $(q>r)$ тогда допусти $r$>3 тогда докажем что 27+$(q^3+r^3)$>$9qr$

$q^3+r^3$+27>2$qr$$\sqrt{qr}$>9$qr$ ведь $\sqrt{qr}$>5 тогда достаточно рассмотреть r=2 и убедится что ответ не походит ведь тогда q=5 или q=7 тогда допустим p=3k+1 тогда допустим q;r не равны 3 тогда i) q=3m+1 и r=3h+1

ii)q=3m+1 и r=3h-1

iii)q=3m-1 и r=3h-1 если рассмотреть все три варианта не трудно убедится что выйдет противоречие тогда Б.О.О q=3 тогда заметим что $r^3+p^3$ делится на 9 тогда $rp^2$ делится на 3 откуда одно из чисел равно 3 тогда допустим

p=3k+2 Лемма: если $a^2-ab+b^2$ делится на

p=3k+2 где оно простое то тогда p | a;b допустим не так тогда по МТФ $ $a^(3k+1)$ \equiv $b^(3k+1)

\pmod {p}$ очевидно

$a^3 \equiv b^3 \pmod {p}$ тогда $a \equiv b \pmod {p}$ откуда p | a;b

Тогда если раскрыть $q^3+r^3$=(q+r)($q^2-qr+r^2$) то q+r=$kp^2$ тогда сократим обе стороны на $p^2$ тогда $p+qrk$=$qr$ что невозможно тогда p=q=r=3

  12
2023-11-23 00:15:14.0 #

Видите, что $(p,q,r)=(3,3,3)$ — решение. Теперь у нас есть $p^2qr=p^3+q^3+r^3\ge 3pqr\Rightarrow p\ge 3$. Если $p=3$, то у нас должно быть равенство, поэтому $q=r=3$ тоже. Предположим, что $p\ge 5$. Легко проверить, что если два числа $p,q,r$ равны, то все они должны быть $3$. Таким образом, $p,q,r$ попарно различны. Мы знаем, что $p^2|(q+r)(q^2+r^2-qr)$. Поскольку $p$ не делит $qr$ и $p\neq 3$, мы знаем, что $V_p(gcd(q+r,q^2+r^2-rq))=0$. Если $p^2|q+r$, то $q+r\ge p^2$. Разделив обе части исходного уравнения на $p^2$, получим $qr=p+(q^2+r^2-rq)\frac{q+r}{p^2}\ge p+q^2+r ^2-qr\ge p+qr$, противоречие. Следовательно, $p^2|q^2+r^2-rq$. В частности, $p\equiv 1\pmod{3}$.

Исследуйте исходное уравнение по модулю $3$. При $x^3\equiv_3 x$ получаем $1+q+r\equiv_3 qr\Rightarrow (q-1)(r-1)\equiv_3 2$. Следовательно, либо $q$, либо $r$ должны быть равны $3$. ВЛОГ $r=3$. Получаем $p^3+q^3+27=3p^2q$. Таким образом, $q|(p+3)(p^2-3p+9)$. Если $q|p+3$, то при $2q=q+q>3+p$ получаем $q=p+3$, что означает, что либо $q$, либо $p$ четно, что невозможно. Тогда $q|p^2-3p+9$. В частности, $q\equiv 1\pmod{3}$. Теперь рассмотрим уравнение $p^3+q^3+27=3p^2q$ по модулю $3$ и получим противоречие: $1+1+27\not\equiv 3\pmod 3$

пред. Правка 2   2
2024-02-25 10:45:07.0 #